Croissants cost 85p each and pain au chocolate cost 95p each. Didier buys 22 pastries and spends a total of £20. How many croissants and how many pain au chocolat did he buy?

Answers

Answer 1

Answer:

croissants: 10        pain au chocolat: 12

Step-by-step explanation:

85c + 95p = 2000

c + p = 22

85c + 95p = 2000    

                               -

85c + 85p = 1870

         10p = 120

           p = 12

c + p = 22

c + 12 = 22

c = 10

Answer 2

Answer:

Step-by-step explanation: Let C be the number of croissants and P be the number of pain au chocolat that Didier buys. We know that C + P = 22 and 0.85C + 0.95P = 20.

We can solve this system of equations using substitution. First, we solve the first equation for P: P = 22 - C. Then, we substitute this expression for P into the second equation: 0.85C + 0.95(22 - C) = 20. Expanding the expression inside the parentheses and combining like terms, we get: 0.85C + 21.1 - 0.95C = 20.

Then, we can solve for C by moving all the terms to one side of the equation: -0.1C = -1.1. Dividing both sides by -0.1, we get: C = 11.

Substituting this value for C back into the equation P = 22 - C, we get: P = 22 - 11 = 11.

Therefore, Didier buys 11 croissants and 11 pain au chocolat.


Related Questions

the spread of a rumor in a town can be modeled as n equals 500 square root of t, where n is the number of people who have heard of the rumor, and t is time (in days). how long will it take until 2000 people know about the rumor? 12 days 4 days 8 days 16 days

Answers

The time it take until 2000 people know about the rumor is 16 days.

The square root of a variety of is described because the value, which offers the variety while it's miles increased with the aid of using itself. The radical symbol √ is used to suggest the square root.  Radical is any other call given to the square root symbol. It is likewise called the surds. While Radicand is the variety gift beneath neath the square root symbol.

Find t, when N=200

N=5OO t^1/2

2000=500 t^1/2

t^1/2=4

t=16 days

Thus, the time it take until 2000 people know about the rumor is 16 days.

To learn more about square root check the link below:

https://brainly.com/question/428672

#SPJ4

Choose the equation of the graphed function.

Answers

Answer:

h

Step-by-step explanation:

because it shifts the graph to the left by 6 units

© {2, 4, 6, 8}
(3, 4, 5, 6}
(2, 3), (4, 4), (6, 5), (8, 6)
{2, 3, 4, 5, 6, 8)
X
2
4
6
8
у
3
4
5
6

Answers

What’s the question?

if the value of x is 3 and the value of y is 5, what is displayed as a result of executing the code segment?

Answers

The result of executing the code segment is -2

How to determine the result of executing the code segment?

The complete question is added at the end of this solution as an attachment

The code in the question is given as

IF X > Y

  DISPLAY X + Y

ELSE

  DISPLAY X - Y

Given that

X = 3 and Y = 5

When x and y are compared, we have the truth value to be

Y > X

This means that the executed segment is

DISPLAY X - Y

So, we have

DISPLAY 3 - 5

Evaluate

DISPLAY -2

Hence. the displayed result is -2

Read more about code segment at

https://brainly.com/question/20734416

#SPJ1

Let P(n) be the statement that 13 +23 +···+n3 = (n(n+1)/2)2 for the positive integer n.a) What is the statement P(1)?b) Show that P(1) is true, completing the basis step of the proof.c) What is the inductive hypothesis?d) What do you need to prove in the inductive step?e) Complete the inductive step, identifying where you use the inductive hypothesis.f) Explain why these steps show that this formula is true whenever n is a positive integer.

Answers

P(n) = 1³ +2³ +···+n³ = (n(n+1)/2)²

P(1) = 1³ = 1

According to statement P(n) = (n(n+1)/2)²

Checking for n = 1,

(n(n+1)/ 2)² = (1(1+1)/ 2)²

P(1) = (1 × 2/ 2)²

P(1) = 1² = 1

P(1) = 1³ = 1

So P(n) holds for n = 1.

The inductive hypothesis is that the statement holds for P(n). To prove this the inductive step under the assumption that the inductive hypothesis is true, we prove it is true for P(n + 1)

Let us assume P(n) = 1³ + 2³ + ... + n³ = (n(n+1)/ 2)²

Then P(n + 1) = 1³ + 2³ + ... + n³ + (n + 1)³ = (n(n+1)/ 2)² + (n + 1)³

P(n + 1) = (n(n+1)/ 2)² + n³ + 3n² + 3n + 1

P(n + 1) = ((n + 1)((n + 1) + 1)/ 2)²

As assuming P(n) holds, it is also true for P(n + 1) so it will be true for all values of n as it is proved already that P(1) is true.

To know more mathematical induction

https://brainly.com/question/29503103

#SPJ4

Linear Functions: Model from Two Points-Quiz-Level H
Ready
4) A bathtub has some water in it. Mia turns on the faucet to add more water. The total amount of
water in gallons, y, is a function of the time in minutes since Mia turns on the faucet, a.
4) The graph of the linear function passes through the points (4, 24) and (6, 30).
What is the equation of the function?
?

Answers

The equation of function for the given problem is y = 3x + 12.

What is point slope form of the line?

For linear equations, the general form is y - y1 = m(x - x1).

It draws attention to the line's slope and one of the line's points (that is not the y-intercept).

Given:

A bathtub has some water in it. Mia turns on the faucet to add more water.

The total amount of water in gallons, y, is a function of the time in minutes since Mia turns on the faucet, a.

The graph of the linear function passes through the points (4, 24) and

(6, 30).

We have to find the equation of function.

Let the linear function passes through the points (4, 24) and (6, 30).

First to find the slope of equation using given points.

[tex]m = \frac{y_2-y_1}{x_2-x_1} = \frac{30-24}{6-4} = \frac{6}{2} = 3[/tex]

Now to find the equation of function.

Consider the point slope form of the line,

[tex]y-y_1=m(x-x_1)[/tex]

Plug the values of m = 3 and [tex](x_1,, y_1) = (4, 24)[/tex]

[tex]y-24=3(x-4)\\y-24=3x-12\\y=3x-12+24\\y=3x+12[/tex]

Hence, the equation of function for the given problem is y = 3x + 12.

To know more about point slope form of line, click on the link

https://brainly.com/question/29797287

#SPJ1

Please help me it’s asap

Answers

Answer:

215

Step-by-step explanation:

218+212/2

if a line equation 2x +y =4 what is the gradie value

Answers

The gradient value of the line 2x + y = 4 is 2. This can be found by rearranging the equation into slope-intercept form, which is y = -2x + 4. The coefficient of the x term, -2, is the slope of the line.

Write a function in any form that would match the graph shown below 

Answers

The function that matches the graph is y(x)=(1-x)(x+2)^2

What is function?

a function from a set X to the  set of Y assigns to each other element of X exactly one  element of Y. The set X is the  called the domain of to the  function of  the set Y is called as  the condominium of the functions.

From the graph the curve cross the x -axis at X=1

Therefore the expression is (x-1)

And at (-) x axis at X=-2 it is the turning point.

Therefore the expression is

(X+2)^2

And the point on y axis is (0,4)

Therefore the function can be written as

Y(x)=a(x-1)(x+2)^2

4= -4a≈a=-1

Therefore the function is y(x)

(-1)(x-1)(x+2)^2

= y(x)=(1-x)(x+2)^2

Therefore the function that matches the graph is y(x)=(1-x)(x+2)^2

To know more about function click-

https://brainly.com/question/25638609

#SPJ1

Solve using a matrix.
2x-6y=22
-5x+y=1



can you please give me something I can copy-paste?

Answers

IT is found that the value of x is 4 and value of y is 5.

What is a system of equations?

A system of equations is two or more equations that can be solved to get a unique solution. the power of the equation must be in one degree.

The given system of equations are

2x-6y=22

-5x+y=1

The matrix form is

[tex]\left[\begin{array}{ccc}2&-6\\-5&1\end{array}\right] \left[\begin{array}{ccc}x\\y\end{array}\right] = \left[\begin{array}{ccc}22\\1\end{array}\right][/tex]

Let as assume

[tex]A = \left[\begin{array}{ccc}2&-6\\-5&1\end{array}\right]\\ \\X = \left[\begin{array}{ccc}x\\y\end{array}\right] \\B = \left[\begin{array}{ccc}22\\1\end{array}\right][/tex]

WE know that AX = B

Then we have;

[tex]\left[\begin{array}{ccc}x\\y\end{array}\right] = \left[\begin{array}{ccc}4\\5\end{array}\right][/tex]

Therefore, the value of x is 4 and value of y is 5.

Learn more about equations here;

https://brainly.com/question/25180086

#SPJ1

Please help me with this

Answers

The distance the ball drops in the next 8 second is: 640 m. Using the concept of equation of motion.

What is equation of motion?

An item is considered to be at rest when its position doesn't alter throughout time. An item is considered to be in motion if, over time, its location changes.

It is possible to build a relationship using a series of equations between the body's velocity, acceleration, and the distance it travels in a given amount of time when the body is travelling in a straight line with uniform acceleration. The term "motion equations" refers to these equations.

D = kt²  where D is the constant of proportionality.

D = 80 m

t = 4 seconds

Putting the values we get -

80 = k(4)²

or, k = 80 / 16

or, k = 5.

So the equation of motion is D = 5t²

When t = 8+4 = 12 seconds

now, D = 5(12)² =  5×144

or, D =  720 m

So, the distance the ball drops in the next 8 seconds = 720 - 80

= 640 m.

To know more about equation of motion refer to:

https://brainly.com/question/27821888

#SPJ1

in class a there are 20 students 14 of them are girls in class b there 25 students 15 of them are girls (a) find the percentages of girls in each class​

Answers

To find the percentage of girls in class A, we need to divide the number of girls by the total number of students and then multiply by 100. This gives us:

14 / 20 * 100 = 70%

To find the percentage of girls in class B, we need to divide the number of girls by the total number of students and then multiply by 100. This gives us:

15 / 25 * 100 = 60%

Thus, the percentage of girls in class A is 70%, and the percentage of girls in class B is 60%.

Given f (x) = -3x² - 6x +9, find f (-7)

Answers

Answer:

f(-7) = - 96

---------------------------------

Given function:

f(x) = -3x² - 6x +9

Find f(-7) by plugging in the value of x:

f(-7) = - 3(-7)² - 6(-7) +9f(-7) = - 3(49) + 42 +9f(-7) = - 147 + 51f(-7) = - 96

Juan claims that y=4x^3+1 is a function, but not a linear function. Select the statement that supports Juan's claim.
A. The function does not contain the point (0,0)
B. The coefficient of x^3 is not 1
C. The graph of the function does not form a straight line
D. The function increases​

Answers

The answer I believe is B???????

Answer:

C. The graph of the function does not form a straight line

The sin (theta) = -2/5, and theta lies in quadrant IV. Find the exact values of the sine and cosine of 2 theta.

Answers

[tex]\displaystyle\\Answer:\ sin(2\theta)=-\frac{4\sqrt{21} }{25} ,\ cos(2\theta)=\frac{17}{25}[/tex]

Step-by-step explanation:

[tex]\displaystyle\\sin(\theta)=-\frac{2}{5} \ \ \ \ \ \ \ \ 270^0 < \theta < 360^0\\\\sin^2(\theta)+cos^2(\theta)=1\\\\cos^2(\theta)=1-sin^2(\theta)\\\\Hence,\\\\cos^2(\theta)=1-(-\frac{2}{5})^2 \\cos^2(\theta)=1-\frac{4}{25} \\\\cos^2(\theta)=\frac{25(1)-4}{25} \\\\cos^2(\theta)=\frac{21}{25} \\\\[/tex]

Extract the square root of both parts of the equation:

[tex]\displaystyle\\cos(\theta)=б\sqrt{\frac{21}{25} } \\\\cos(\theta)=б\frac{\sqrt{21} }{5} \\\\270^0 < \theta < 360^0\\\\Hence,\\\\cos(\theta)=\frac{\sqrt{21} }{5}[/tex]

[tex]\displaystyle\\a)\ sin(2\theta)=2sin(\theta)cos(\theta)\\\\sin(2\theta)=2(-\frac{2}{5})(\frac{\sqrt{21} }{5})\\\\sin(2\theta)=-\frac{4\sqrt{21} }{25}[/tex]

[tex]\displaystyle\\b)\ cos(2\theta)=cos^2(\theta)-sin^2(\theta)\\\\cos(2\theta)=(\frac{\sqrt{21} }{5})^2-(-\frac{2}{5})^2 \\\\cos(2\theta)=\frac{21}{25}-\frac{4}{25} \\\\cos(2\theta)=\frac{17}{25}[/tex]

Select the graph of the solution. Click until the correct graph appears.

|x| + 1 < 3

Answers

Answer:

Graph B

Step-by-step explanation:

First, simplify by subtracting 1 on both sides.

|x| + 1 - 1 < 3 - 1

|x| < 2

Using the absolute value definition, we know the inequalities are:

x < 2

-x < 2

Divide both sides by -1.

x < -2

If you multiply or divide both sides of an inequality by a negative number, you must flip the sign.

x > -2

x<2, x>-2

Graph B

Answer:

Graph A

Step-by-step explanation:

if x ≥ 0

x + 1 < 3

x < 3 - 1

x < 2

0≤ x < 2


if x < 0

-x + 1 < 3

-x < 3 - 1

-x < 2

x > -2


-2 < x < 0

Final solution

-2 < x < 2

The figure shows a loading dock and a side view of
an attached ramp, whose run is 12 feet and whose
rise is 39 inches. Joaquin is wondering whether a
long rectangular box can be stored underneath the
ramp, as suggested by the dotted lines. The box is
2 feet tall and 5 feet long. Answer Joaquin's question.

Answers

The area of the triangular dock is 19.5 square feet and the area of the rectangular box is 10 feet. The box will fit under the dock.

What is an area?

The space occupied by any two-dimensional figure in a plane is called the area. The space occupied by the rectangle in a two-dimensional plane is called the area of the rectangle. The space occupied by the triangle in a two-dimensional plane is called the area of the triangle.

Calculate the area of the triangle first,

Area = 1 / 2 x B x H

Area = 1/2 x 12 x 3.25

Area = 19.5 square feet

Calculate the area of the rectangle,

Area = L x W

Area = 2 x 5

Area = 10 square feet

To know more about an area follow

https://brainly.com/question/29223290

#SPJ1

Use the marginal tax rate chart to answer the question.
Marginal Tax Rate Chart
Tax Bracket
Marginal Tax Rate
$0-$10,275
10%
$10.276-$41.175
12%
$41.176-$89.075
22%
$89,076-$170.050 24%
$170,051-$215,950 32%
$215,951-$539,900 35%
$539,901
37%
Determine the amount of taxes owed on a taxable income of $49,652.
Answer choices
$4,735.38
$6,600.44
$7,709.92
$10,293.44

Answers

The amount of taxes owed on a taxable income of $49,652 is $10,293.44.

What is the percentage?

A percentage is a value per hundredth. Percentages can be converted into decimals and fractions by dividing the percentage value by a hundred.

Given, Are the tax percentages for various amounts of income.

Now, From the given chart we conclude that $49,652 falls in the category of $41.176-$89.075 is 22% tax.

So, the Tax owed is 22% of $49,652.

(22/100)×$49,652.

= $10,293.44.

So, an income of $49,652 owed 22% tax of the income which is $10,293.44.

learn more about percentages here :

https://brainly.com/question/24159063

#SPJ1

Answer:THE ANSWER IS B)6,600.44

Step-by-step explanation:

I got it right :)

IN PHOTO⬇️
PLEASE HELP!!!

Answers

The simple interest is $2160 and the amount after 30 years is $6160.

Given that, principal =$4000, rate of interest =1.8%, and time period =30 years.

What is the simple interest?

Simple interest is a method to calculate the amount of interest charged on a sum at a given rate and for a given period of time.

Simple interest is calculated with the following formula: S.I. = (P × R × T)/100, where P = Principal, R = Rate of Interest in % per annum, and T = Time, usually calculated as the number of years.

a) Now, S.I =(4000×1.8×30)/100

= 40×1.8×30

= $2160

b) Amount = Principal + Interest

= 4000 + 2160

= $6160

Therefore, the simple interest is $2160 and the amount after 30 years is $6160.

To learn more about the simple interest visit:

https://brainly.com/question/25845758.

#SPJ1

The acute angle between the vectors a=i-kj and b=i+jis 60° Calculate the possible values of k

no clue how to reach the answer ​

Answers

Answer:

k = (-55) / 8

k = (-3005) / 8

k = (-255 - sqrt(65025 - 510((-255 + sqrt(65025 - 510((-255 + sqrt(65025 - 510(0.309016^2))) / 2)^2)) / 2)^2)) / 2

k = (-255 - sqrt(65025 - 510((-255 + sqrt(65025 - 510((-255 + sqrt(65025 - 1469.59)))))^2)) / 2)

To find the acute angle between two vectors, we can use the dot product formula:

angle = arccos((a * b) / (||a|| * ||b||))

where a and b are the vectors, * is the dot product, and ||a|| and ||b|| are the magnitudes of the vectors a and b, respectively.

In this case, the dot product of a and b is (i - kj) * (i + j) = i^2 - kj * i + kj * i + kj^2 = 2i - k^2j

The magnitudes of the vectors a and b are ||a|| = sqrt(i^2 + (-kj)^2) = sqrt(1 + k^2) and ||b|| = sqrt(i^2 + j^2) = sqrt(2).

Substituting these values into the formula above, we get:

angle = arccos((2i - k^2j) / (sqrt(1 + k^2) * sqrt(2)))

Since the angle is given to be 60 degrees, we can set this equal to 60 degrees and solve for k:

60 = arccos((2i - k^2j) / (sqrt(1 + k^2) * sqrt(2)))

We can use the inverse cosine function to solve for k:

k = sqrt(1 / (cos(60)^2 - (2i / sqrt(1 + k^2) * sqrt(2))^2))

Since cos(60) = 0.5, we can substitute this value in and solve for k:

k = sqrt(1 / (0.5^2 - (2i / sqrt(1 + k^2) * sqrt(2))^2))

k = sqrt(1 / (0.25 - (2i / sqrt(1 + k^2) * sqrt(2))^2))

k = sqrt(1 / (0.25 - (4i^2 / (1 + k^2) * 2)^2))

k = sqrt(1 / (0.25 - (16 / (1 + k^2))^2))

k = sqrt(1 / (0.25 - 256 / (1 + k^2)^2))

k = sqrt((1 + k^2)^2 / (256 - (1 + k^2)^2))

k = sqrt((1 + k^4) / (256 - 1 - 2k^2 - k^4))

k = sqrt((k^4 + 1) / (255 - 2k^2))

We can then solve for the roots of this equation to find the possible values of k:

k = sqrt((k^4 + 1) / (255 - 2k^2))

k^4 - (255 - 2k^2)k^2 + 1 = 0

This is a quartic equation and can be solved using the quartic formula:

k = sqrt((-b +- sqrt(b^2 - 4ac)) / 2a)

where a, b, and c are the coefficients of the polynomial. In this case, a = 1, b = -(255 - 2k^2), and c = 1.

Substituting these values into the quartic formula, we get:

k = sqrt((-(-(255 - 2k^2)) +- sqrt((-(255 - 2k^2))^2 - 4 * 1 * 1)) / 2 * 1)

k = sqrt((255 - 2k^2 +- sqrt((255 - 2k^2)^2 - 4)) / 2)

k = sqrt((255 - 2k^2 +- sqrt(255^2 - 510k^2 + 4k^4)) / 2)

k = sqrt((255 - 2k^2 +- sqrt(255^2 - 510k^2)) / 2)

k = sqrt((255 - 2k^2 +- sqrt(65025 - 510k^2)) / 2)

Solving for the roots of this equation gives us the possible values of k:

k = (-255 + sqrt(65025 - 510k^2)) / 2

k = (-255 - sqrt(65025 - 510k^2)) / 2

The first equation gives us one possible value of k:

k = (-255 + sqrt(65025 - 510k^2)) / 2

Substituting k = (-255 + sqrt(65025 - 510k^2)) / 2 into the second equation gives us the second possible value of k:

k = (-255 - sqrt(65025 - 510((-255 + sqrt(65025 - 510k^2)) / 2)^2)) / 2

Simplifying this expression gives us the final possible value of k:

k = (-255 - sqrt(65025 - 510((-255 + sqrt(65025 - 510((-255 + sqrt(65025 - 510k^2)) / 2)^2)) / 2)^2)) / 2

Therefore, the possible values of k are:

k = (-255 + sqrt(65025 - 510k^2)) / 2

k = (-255 - sqrt(65025 - 510((-255 + sqrt(65025 - 510k^2)) / 2)^2)) / 2

solve for k in each

To solve for k in the first equation, we can isolate k by moving everything else to the right side of the equation:

k = (-255 + sqrt(65025 - 510k^2)) / 2

2k = -255 + sqrt(65025 - 510k^2)

2k + 255 = sqrt(65025 - 510k^2)

(2k + 255)^2 = 65025 - 510k^2

4k^2 + 1020k + 65025 = 65025 - 510k^2

4k^2 + 1530k + 65025 = 0

This is a quadratic equation, and we can use the quadratic formula to solve for k:

k = (-b +- sqrt(b^2 - 4ac)) / 2a

where a, b, and c are the coefficients of the polynomial. In this case, a = 4, b = 1530, and c = 65025.

Substituting these values into the quadratic formula gives us:

k = (-1530 +- sqrt(1530^2 - 4 * 4 * 65025)) / 2 * 4

k = (-1530 +- sqrt(3080400 - 2601000)) / 8

k = (-1530 +- sqrt(477900)) / 8

k = (-1530 +- sqrt(222725)) / 8

k = (-1530 + 1475) / 8

k = (-55) / 8

k = (-1530 - 1475) / 8

k = (-3005) / 8

Therefore, the solutions to the first equation are:

k = (-55) / 8

k = (-3005) / 8

how many additional groups would be required to conduct a 3 x 2 x 3 factorial design compared to a 3 x 2 x 2 design?

Answers

One additional group would be required to conduct a 3 x 2 x 3 factorial design compared to a 3 x 2 x 2 design with independent variable.

A 3 x 2 x 3 factorial design requires three independent variables (x1, x2, and x3) with three levels each, for a total of 27 conditions. A 3 x 2 x 2 design, however, would only require two independent variables (x1 and x2) with two levels each, for a total of 12 conditions. To conduct a 3 x 2 x 3 factorial design, one additional group would be required, compared to the 3 x 2 x 2 design. This additional group would provide data for the additional 15 conditions that the 3 x 2 x 3 design would require.

Learn more about independent variable here

https://brainly.com/question/29430246

#SPJ4

Find the value of k that would make the left side of the equation a perfect square trinomial.
9x² - kx + 4 = 0

Answers

Answer: k=±`12

Step-by-step explanation:

[tex]ax^2-bx+c=0\\\\9x^2-kx+4=0\\\\D=b^2-4ac\\\\a=9\ \ \ \ b=-k\ \ \ \ c=4\\\\D=0\\\\Hence,\\\\D=(-k)^ 2-4(9)(4)=0\\\\k^2-144=0\\\\k^2=144\\\\k^2=12^2[/tex]

Extract the square root of both parts of the equation:

[tex]k=б\sqrt{12^2} \\\\k=б12[/tex]

Answer:

k = - 12 or k = 12

---------------------------------------------

A perfect square trinomial is:

(a ± b)² = a² ± 2ab + b²

Given trinomial:

9x² - kx + 4 = 0

Show this as:

(±3x)² - kx + (±2)² = 0

Compare and find possible values of k:

k = - 2(±3)(±2) = ± 12

as the sample size increases, the distribution of the sample proportion becomes more normal. this fact is due to

Answers

the Central Limit Theorem, which states that as the sample size increases, the sampling distribution of the sample mean tends to become more normal.

The Central Limit Theorem states that as the sample size increases, the sampling distribution of the sample mean tends to become more normal. This means that the mean of the sample will be closer to the mean of the population, and the variability of the sample will be smaller. Since the sample proportion is just the mean of the sample, as the sample size increases, the distribution of the sample proportion will also become more normal. The larger the sample size, the more likely it is that the sample will accurately represent the population, and the more normal the distribution of the sample proportion will be. In other words, the sample proportion will be closer to the true population proportion, and the variability of the sample proportion will be smaller.

Learn more about central limit theorem here

https://brainly.com/question/18403552

#SPJ4

a company has the following information regarding its forecast performance in the past three periods. icture what is the mean absolute deviation (mad)? question 26 options: 225 -66.7 1200 200

Answers

The mean absolute deviation over three period of time is 200

The absolute value of error in period 1 = 300

The absolute value of error in period 2 = 200

The absolute value of error in period 3 = -100

Total absolute value of error = The absolute value of error in period 1 + The absolute value of error in period 2 + The absolute value of error in period 3

Substitute values in the equation

Total absolute value of error  = 300 + 200 + 100

= 600

The mean absolute deviation = Total absolute value of error / 3

Substitute the values in the equation

The mean absolute deviation  = 600 / 3

= 200

Therefore, the mean absolute deviation is 200

Learn more about mean absolute deviation here

brainly.com/question/10528201

#SPJ4

It takes 73 pounds of seed to completely plant an 11 -acre field. How many pounds of seed are needed per acre?

Answers

Step-by-step explanation:

It takes 73 pounds of seed to completely plant an 11 -acre field. How many pounds of seed are needed per acre?

73 / 11 = 6.6363636 pounds per acre

rounded to two decimal places: 6.64 pounds per acre

Lena's phone is 15 cm long and 8 cm wide. Her tablet is 30 cm long and 22 cm wide.
How much additional area is available on her tablet than on her phone?

Answers

Step-by-step explanation:

subtract the area of the phone from that of the phone

A number is such that it is as much greater than 112 as it is less than it. find the number.​

Answers

Answer:

Answer. it is -111 because it is greater than 112 and in positive form it is smaller and in negative it is bigger .

11. Define the range of the following:

Answers

Answer:

The answer is B.

Step-by-step explanation:

Looking at the points gives you a x-axis and y-axis.

(x , y)

(-2 , 3) ,

(0 , 5) ,

(1 , 5) ,

(2 , 4) ,

(8 , -2)

Focus on the y-axis instead of the x-axis and list them.

{3 , 5 , 4 , -2}

What is the value of y when x = 3 in the equation y = 6x - (9 + 5)

Answers

Answer:

y = 4

Step-by-step explanation:

substitute x = 3 into the equation

y = 6(3) - (9 + 5) = 18 - 14 = 4

Answer: y=4

Step-by-step explanation:

y=6(3)-(9+5)

y=18-14

y=4

Is the line perpendicular?
The situation is based on football. One player starts a couple of yards in the endzone while the other starts at the 8-9 yard line. The player in the endzone almost scores when he is tracked down by the guy on the 8-9 yard line. So does the starting position of these players form a perpendicular line?

Answers

Yes, the starting position of these players form a perpendicular line.

What is Trigonometry?

The area of mathematics that deals with particular angles' functions and how to use those functions in calculations. There are six popular trigonometric functions for an angle. Sine (sin), cosine (cos), tangent (tan), cotangent (cot), secant (sec), and cosecant are their respective names and acronyms (csc).

Given:

Using Trigonometry

sin [tex]\theta[/tex] = 33.33/100

sin [tex]\theta[/tex] = 0.3333

[tex]\theta[/tex] = [tex]sin^{-1}[/tex] (0.3333)

[tex]\theta[/tex] = 19.469

cos 19.469 = B/ 100

0.9428 = B/ 100

B= 94.28

tan  [tex]\theta[/tex] = P/B

tan  [tex]\theta[/tex] = 94.28/ 33.33

[tex]\theta[/tex] = 70.5

Using Angle Sum property

< 3= 180 - (70.5 + 19.5)

<3 = 180 - 90

<3 = 90

Hence, they form perpendicular line.

Learn more about Trigonometry here:

https://brainly.com/question/29002217

#SPJ1

Other Questions
Select the correct answer from each drop-down menu.Chris in lives in a city that experiences heavy rainfall with average annual precipitation of 882 millimeters. It is warm all year.Mark lives in city with an average annual rainfall of 40 millimeters. It has hot summers and cool winters.Chris's city is in aclimate, while Mark's city is in aclimate.ResetNeed denny corporation is considering replacing a technologically obsolete machine with a new state-of-the-art numerically controlled machine. the new machine would cost $290,000 and would have a ten-year useful life. unfortunately, the new machine would have no salvage value. the new machine would cost $48,000 per year to operate and maintain, but would save $89,000 per year in labor and other costs. the old machine can be sold now for scrap for $29,000. the simple rate of return on the new machine is closest to (ignore income taxes.): according to the dietary guidelines for americans, adults should consume between - % total dietary fat, and less than % saturated fat. which of the following is not a gap that makes up the service gaps model? a. knowledge b. delivery c. tangibility d. design and standards e. communication TRUE OR FALSE : in an online speech you have essentially the same relationship with your audience as in an in-person speech. group starts Crane Company combines its operating expenses for budget purposes in a selling and administrative expense budget. For the first 6 months of 2018, the following data are available.1. Sales: 20,800 units quarter 1; 22,100 units quarter 2.2. Variable costs per dollar of sales: sales commissions 5%, delivery expense 2%, and advertising 3%.3. Fixed costs per quarter: sales salaries $10,900, office salaries $6,160, depreciation $4,490, insurance $2,080, utilities $880, and repairs expense $670.4. Unit selling price: $24. samantha experienced a traumatic brain injury and afterward began to exhibit bizarre symptoms that no one had ever documented before. the best research method to study samanta would be what is the target of thyroid hormones? what is the target of thyroid hormones? thyroid anterior pituitary hypothalamus cells of the body The increased presence of user operated computers in the workplace has resulted in an increasing number of persons having access to the system. A control that is often used tp prevent unauthorized access to sensitive programs is:A) Backup of data in the cloud.B) Authentication procedures.C) input validation checks.D) Record counts of the number of input transactions in a batch being processed. 43. Ford Davis borrowed $14,000 of a non-interest-bearing, simple discount, 8% , 90-day note. Assume ordinary interest. What are Ford's proceeds?A. $14,000B. $13,720C. $13,000D. $17,320 for a random sample of 90 such pairs, where is the sampling distribution of x centered, and what is the standard deviation of the x distribution? use the formulas for lowering powers to rewrite the expression in terms of the first power of cosine, as in example 4. cos4(x) sin2(x) hi hi please help metalk about the quote and explain it in 4-5 sentences The world become flesh and lived among us. John 1:14 in early christianity artists incorporated motifs or figures from pagan cultures into christian artwork. Caleb wa out at a retaurant for dinner when the bill came. Hi dinner came to $21. He wanted to leave a 31% tip. How much wa hi meal plu the tip, before tax, in dollar and cent the volume of a speherical baloon is increasing at a constant rate of 100cm^3/s. how fast is the baloon's radius increasing when its diamter is 50 cm g carpal tunnel syndrome is a condition in which the median nerve is compressed as it travels through a narrow passage in the wrist. explain how hand dynamometry might be used to assess a patient's need for surgery to release carpal tunnel compression. FILL IN THE BLANK. Three commonly used approaches to cloud computing are public cloud computing, private cloud computing, and ___________ cloud computing. which statement about the nucleus is incorrect? a-the nucleus contains the instructions for copying itself b-the nucleus contains the genetic material of the cell c-the nucleus contains the information that ribosomes use to carry out cellular respiration d-the nucleus contains the information that ribosomes use to carry out protein synthesis a. A mass, m2, is initially at rest. A mass, m1, traveling with velocity, v1, collides with m2 in an elastic collision. After the collision, the two objects move with final velocities, v1f and v2f respectively. Using the laws of conservation of energy and momentum, show that the ratio of the final velocities in terms of m1 and m2 is given by: (v2f/v1f) = (2m1/m1 m2) b. Linearize the expression in Exercise 1 by making the following substitutions: ym2/m1 andxv1f/v2f If y is plotted as a function of x, what values do you expect for the slope and intercept? c. Based on your result from Exercise2, what ratio of masses, m2/m1,results in final velocities, v1f andv2f that are equal in magnitude and opposite in direction?